Difference between revisions of "2021 AMC 12B Problems/Problem 25"

Line 1: Line 1:
 +
==Problem==
 +
 +
Let <math>S</math> be the set of lattice points in the coordinate plane, both of whose coordinates are integers between <math>1</math> and <math>30,</math> inclusive. Exactly <math>300</math> points in <math>S</math> lie on or below a line with equation <math>y=mx.</math> The possible values of <math>m</math> lie in an interval of length <math>\frac ab,</math> where <math>a</math> and <math>b</math> are relatively prime positive integers. What is <math>a+b?</math>
 +
 +
<math>\textbf{(A) }31 \qquad \textbf{(B) }47 \qquad \textbf{(C) }62\qquad \textbf{(D) }72 \qquad \textbf{(E) }85</math>
 +
 
{{AMC10 box|year=2021|ab=B|num-b=24|after=Last Problem}}
 
{{AMC10 box|year=2021|ab=B|num-b=24|after=Last Problem}}

Revision as of 02:51, 12 February 2021

Problem

Let $S$ be the set of lattice points in the coordinate plane, both of whose coordinates are integers between $1$ and $30,$ inclusive. Exactly $300$ points in $S$ lie on or below a line with equation $y=mx.$ The possible values of $m$ lie in an interval of length $\frac ab,$ where $a$ and $b$ are relatively prime positive integers. What is $a+b?$

$\textbf{(A) }31 \qquad \textbf{(B) }47 \qquad \textbf{(C) }62\qquad \textbf{(D) }72 \qquad \textbf{(E) }85$

2021 AMC 10B (ProblemsAnswer KeyResources)
Preceded by
Problem 24
Followed by
Last Problem
1 2 3 4 5 6 7 8 9 10 11 12 13 14 15 16 17 18 19 20 21 22 23 24 25
All AMC 10 Problems and Solutions